3
  1. Does electron's magnetic spin mean that we can think of it as having the two magnetic poles?

  2. Does the spin mean that it generates magnetic field even if it is at rest?

I expect to get 'no' to both questions because we always say moving charges generate magnetic field. And I hope to see answers which will clear my confusion.

Emilio Pisanty
  • 132,859
  • 33
  • 351
  • 666
physicsguy19
  • 1,492
  • 10
  • 28
  • 1
    Possible duplicate of https://physics.stackexchange.com/questions/9969/does-a-magnetic-field-arise-from-a-moving-charge-or-from-its-spin-or-both?rq=1 but I wrote a short answer, as it seems more appropriate for the OP to learn more from. (Hopefully) –  Dec 02 '18 at 22:25
  • 1
    @physicsguy19: the answer by Hans de Vries present in the link cited by StudyStudyStudy contains the answer to your question. n.2. The wavefunction of a charged particle with a non-zero spin carries a current which explains the magnetic dipole of the electron. – GiorgioP-DoomsdayClockIsAt-90 Dec 02 '18 at 23:03
  • Thank you. Actually I'm so happy to get a 'yes' for both questions I love this site. – physicsguy19 Dec 03 '18 at 06:01

2 Answers2

7

Does electron's magnetic spin mean that we can think of it as having the two magnetic poles?

Yes. There are no known magnetic monopoles, see: Magnetic Monopoles

Does the spin mean that it generates magnetic field even if it is at rest?

Yes, but as regards "at rest", you should read this answer: Electron at rest

Magnetic spin isn't really a proper term, but an electron's spin is directly linked to its magnetic field, if we were to say that only moving electric charges produce magnetic fields, we would have trouble explaining how a static magnetic "works".

I would also stress that spin in this case is purely a mathematical idea, an electron does not resemble a very small rotating football in this case, its a bad choice of words made years ago.

If you read the links, you will get a better answer than this short reply.

5

The electron can be seen as a small magnet with the field of a magnetic dipole of strength $2\mu_B \vec S$. So my answer is yes in both cases. I interpret as at rest a state in which the electron has zero momentum expectation value. Note that there is no accepted model of spin as a rotation, although spin is an angular moment to every effect.

Emilio Pisanty
  • 132,859
  • 33
  • 351
  • 666
my2cts
  • 24,097
  • does it mean that when an electron is released (with zero speed classically) in a uniform magnetic field, it just aligns its magnetic moment with the field and does no translational motion (classically)? – physicsguy19 Dec 03 '18 at 08:05
  • Yes, that is correct. – my2cts Dec 03 '18 at 08:22
  • @my2cts: which one is correct?:) sorry.. – physicsguy19 Dec 03 '18 at 08:47
  • My comment was on your question. – my2cts Dec 03 '18 at 09:21
  • 1
    @StudyStudy No motion or translation will occur in a uniform field. – my2cts Dec 03 '18 at 09:23
  • @my2cts. Thanks, 2am (wrong) answer:) https://courses.lumenlearning.com/boundless-physics/chapter/motion-of-a-charged-particle-in-a-magnetic-field/ –  Dec 03 '18 at 11:09
  • One last question please, if an electron is released (zero initial speed) near a U shaped magnet's North pole, does it start moving? I mean if we were to put a tiny magnet there, it would start moving and stick to the north pole. But for an electron the force e(vxB) and the v is zero.. – physicsguy19 Dec 03 '18 at 12:21
  • 1
    It will move as the field is nonuniform. In your original question you assumed a uniform field. – my2cts Dec 03 '18 at 12:59
  • what about e(vxB) then? how can it start moving when v=0? – physicsguy19 Dec 03 '18 at 13:50
  • Good point. Only if motion of charge is underlying the magnetic dipole you can apply the Lorentz force. – my2cts Dec 03 '18 at 13:53
  • when we model the electron as a loop of current, this explains the motion in space varying magnetic field. And also in the link studystudy shared in his post, there is a current like part in the spin magnetic moment equation for the electron. This makes sense. Am I correct? – physicsguy19 Dec 03 '18 at 15:35